Area Inside an Obtuse Triangle

Geometry Level 4

Triangle A B C ABC has an obtuse angle at B B , base B C BC has length equal to 30 30 and height equal to 24 24 . (This height is taken with respect to base B C BC ). D D is a point on the line segment B C BC and E E is a point on A C AC such that D E A B DE \parallel AB . F F is a point on A B AB such that F D A C FD \parallel AC . As D D varies within line segment B C BC , what is the maximum value of [ D E F ] [DEF] ?

Details and assumptions

[ P Q R S ] [PQRS] denotes the area of the figure P Q R S PQRS .


The answer is 90.

This section requires Javascript.
You are seeing this because something didn't load right. We suggest you, (a) try refreshing the page, (b) enabling javascript if it is disabled on your browser and, finally, (c) loading the non-javascript version of this page . We're sorry about the hassle.

18 solutions

Bob Smith
May 20, 2014

From the problem, we can see that A E D F AEDF is a parallelogram. Hence, [ D E F ] = [ A E D F ] / 2 [DEF] = [AEDF] / 2 . Next, note that C E D C A B CED \sim CAB and B D F B C A BDF \sim BCA . Furthermore, since B D + D C = B C BD+DC=BC , their proportionality constants add up to 1. Let them be x x and 1 x 1-x respectively. Then, [ A E D F ] = [ A B C ] [ B D F ] [ C D E ] = [ A B C ] ( 1 x 2 ( 1 x ) 2 ) [AEDF] = [ABC] - [BDF] - [CDE] = [ABC] (1-x^2-(1-x)^2) . By Power Mean Inequality, x 2 + ( 1 x ) 2 1 / 2 x^2+(1-x)^2 \geq 1/2 . Thus, [ A E D F ] [ A B C ] / 2 [AEDF] \leq [ABC] / 2 , and [ D E F ] [ A B C ] / 4 [DEF] \leq [ABC] / 4 . We may easily compute [ A B C ] = 360 [ABC] = 360 , so [ D E F ] 90 [DEF] \leq 90 . Equality is attained when 1 x = x , x = 1 / 2 1-x = x, x=1/2 , and D D is selected to be the midpoint of B C BC . D E F DEF is known as the medial triangle of A B C ABC .

There were numerous approaches, all based on the idea of looking at ratio of B D B C \frac {BD}{BC} . Some students who realized that [ D E F ] = [ A F E ] = A F × F E × sin α [DEF]=[AFE] = AF \times FE \times \sin \alpha could directly argue that we want to maximize B D ( 30 B D ) 3 0 2 \frac {BD(30-BD)}{30^2} .

Note that the condition of obtuse triangle isn't necessary, though it gives more interesting diagrams to look at.

Calvin Lin Staff - 7 years ago
Joshua Xiong
May 20, 2014

We note that the area of a triangle is a b sin C 2 \frac{ab\,\sin C}{2} , so [ A B C ] = ( A B ) ( A C ) sin B A C 2 = 360 [ABC]=\frac{(AB)(AC)\, \sin\angle BAC}{2}=360 and

[ D E F ] = ( D E ) ( D F ) sin F D E 2 [DEF]=\frac{(DE)(DF)\,\sin \angle FDE}{2} . Thus, it suffices to maximize

360 [ D E F ] [ A B C ] = 360 D E A B D F A C sin F D E sin B A C ( 1 ) 360\cdot\frac{[DEF]}{[ABC]} =360\cdot\frac{DE}{AB}\cdot\frac{DF}{AC}\cdot\frac{\sin\angle FDE}{\sin\angle BAC}\quad(1) .

Since D E A B DE\parallel AB and D F A C DF\parallel AC , we know that A E D F AEDF is a parallelogram, and F D E = B A C sin F D E = sin B A C \angle FDE=\angle BAC\implies\sin\angle FDE=\sin\angle BAC . Furthermore, B F D B A C \triangle BFD\sim\triangle BAC and C D E C B A \triangle CDE\sim\triangle CBA . Let us set B D = x BD=x , which in turn, implies that C D = 30 x CD=30-x . Using these similar triangles, we can see that D F A C = x 30 \frac{DF}{AC}=\frac{x}{30} and D E A B = 30 x 30 \frac{DE}{AB}=\frac{30-x}{30} .

When we substitute these ratios into ( 1 ) (1) , we see that we need to maximize 360 30 x 30 x 30 1 = 2 x 2 + 60 x 5 360\cdot\frac{30-x}{30}\cdot\frac{x}{30}\cdot1=\frac{-2x^2+60x}{5} . The maximum value of this quadratic expression occurs at x = 60 2 ( 2 ) = 15 x=-\frac{60}{2(-2)}=15 . We note that at the maximum, D D is the midpoint of B C {BC} , D E A B DE\parallel AB , and D F A C DF\parallel AC , so D E F \triangle DEF is the medial triangle of A B C \triangle ABC , and has 1 4 \frac{1}{4} the area, so max ( [ D E F ] ) = 1 4 360 = 90 \max([DEF])=\frac{1}{4}\cdot 360=\boxed{90} .

Abhishek Sinha
May 20, 2014

First, draw the figure and determine the angles. Now assume D C = x , B D = b x DC=x, BD=b-x , where b = B C = 30 b=BC=30 . From D E C \triangle DEC , we can write E D = sin C sin A x ED=\frac{\sin{C}}{\sin{A}}x and from F D B \triangle FDB we can write F D = sin B sin A ( b x ) FD=\frac{\sin{B}}{\sin{A}}(b-x) . Hence [ D E F ] = 1 2 D E × D F sin F D E [DEF]=\frac{1}{2}DE\times DF \sin \angle FDE = 1 2 sin C sin B sin A x ( b x ) =\frac{1}{2}\frac{\sin{C}\sin{B}}{\sin{A}}x(b-x)

Clearly, [ D E F ] [DEF] is maximized at x = b 2 = 15 x=\frac{b}{2}=15 . And [ D E F ] max [DEF]_{\max} = b 2 8 1 cot C + cot B =\frac{b^2}{8}\frac{1}{\cot{C}+\cot{B}} (by writing sin A = sin ( B + C ) \sin{A}=\sin({B+C}) and dividing the Dr. by the Nr.) = 30 × 24 8 = 90 = \frac{30\times 24}{8}=90

Avi Levy
May 20, 2014

Let a = B D B C a=\frac{BD}{BC} . Since D E F DEF is congruent to A F E AFE , we have [ A B C ] = [ F B D ] + 2 [ D E F ] + [ E D C ] [ABC]=[FBD]+2[DEF]+[EDC] , which implies that [ D E F ] = ( 1 a 2 ( 1 a ) 2 2 ) [ A B C ] = a ( 1 a ) [ A B C ] [DEF]=\left(\frac{1-a^2-(1-a)^2}{2}\right)[ABC]=a(1-a)[ABC]

As D D varies on B C BC , this allows a a to vary in [ 0 , 1 ] [0,1] while [ A B C ] [ABC] remains fixed. Thus the maximum occurs when a ( 1 a ) a(1-a) is maximized, which is at a = 1 2 a=\frac{1}{2} . Then [ D E F ] = 1 4 [ A B C ] = 90 [DEF]=\frac{1}{4}[ABC]=90

Bill Huang
May 20, 2014

We let the ratio BD/BC be r. To maximize the area of DEF, we will find the optimal value of r. Note that triangle EDC is similar to triangle ABC. We then have that ED/DC = AB/BC, which yields that DE = (1-r)AB. We now drop a perpendicular from B to DE, and call the intersection G. Note that BGD will stay the same shape no matter where D is chosen. If we let BG/BD = k, we then have that BG = (kr)BC. The area of triangle DEF, from the base-height formula, is DE BG/2 = AB BC kr(1-r)/2. Since AB, BC, and k are fixed, we need to maximize r(1-r), which occurs at r = 1/2. Thus, D is the midpoint of BC. From the midpoint-parallel line theorem, E and F are the midpoints of AC and AB, respectively. Note that triangles AFE, DEF, FBD, and EDC are all congruent. Thus, the area of DEF is one-fourth that of ABC. The area of ABC, using the base-height formula, is 30 24/2 = 360. Dividing by four to get the area of DEF yields 90.

Yang Conan Teh
May 20, 2014

First, notice that [ D E F ] [\bigtriangleup DEF] = [ A F D E ] 2 \frac{[ AFDE]}{2} . So as we know that [ A B C ] = 24 × 30 2 = 360 , [\bigtriangleup ABC]=\dfrac{24\times 30}{2}=360, it remains to find the minimum of [ F B D ] + [ E D C ] [\bigtriangleup FBD]+[\bigtriangleup EDC] .

First, since D E A B , F D A C DE\parallel AB, FD\parallel AC , we know that C E C A = C D C B , B D B C = B F B A . \dfrac{CE}{CA}=\dfrac{CD}{CB}, \dfrac{BD}{BC}=\dfrac{BF}{BA}. So [ B F D ] = ( B D B C ) 2 × [ A B C ] = 360 ( B D 30 ) 2 . [\bigtriangleup BFD]=(\dfrac {BD}{BC})^{2}\times [\bigtriangleup ABC]=360(\dfrac {BD}{30})^{2}. Similarly, [ C E D ] = ( C D B C ) 2 × [ A B C ] = 360 ( 30 B D 30 ) 2 , [\bigtriangleup CED]=(\dfrac {CD}{BC})^{2}\times [\bigtriangleup ABC]=360(\dfrac {30-BD}{30})^{2}, since C D = B C B D = 30 B D . CD=BC-BD=30-BD. So [ F B D ] + [ E D C ] = 360 ( ( B D 30 ) 2 + ( 30 B D 30 ) 2 ) . [\bigtriangleup FBD]+[\bigtriangleup EDC]=360((\dfrac {BD}{30})^{2}+(\dfrac {30-BD}{30})^{2}).

Now we investigate the minimum of ( B D 30 ) 2 + ( 30 B D 30 ) 2 (\dfrac {BD}{30})^{2}+(\dfrac {30-BD}{30})^{2} . Obseve that by QM-AM inequality, since both B D , 30 B D 0 , BD, 30-BD\ge 0, , we have: ( B D ) 2 + ( 30 B D ) 2 2 B D + 30 B D 2 = 15 , \sqrt{\dfrac{{(BD)^{2}+(30-BD)^{2}}}{2}}\ge\dfrac {BD+30-BD}{2}=15, so ( B D 2 + ( 30 B D ) 2 1 5 2 × 2 = 450 (BD^{2}+(30-BD)^{2}\ge 15^{2}\times 2=450 , with equality holds if and only if B D = 15 BD=15 . So ( B D 30 ) 2 + ( 30 B D 30 ) 2 450 900 = 0.5 , (\dfrac {BD}{30})^{2}+(\dfrac {30-BD}{30})^{2}\ge \frac{450}{900}=0.5, with equality holds iff B D = 15 BD=15 .

Finally, [ F B D ] + [ E D C ] = 360 ( ( B D 30 ) 2 + ( 30 B D 30 ) 2 ) 360 × 0.5 = 180 , [\bigtriangleup FBD]+[\bigtriangleup EDC]=360((\frac {BD}{30})^{2}+(\frac {30-BD}{30})^{2})\ge 360\times 0.5=180, we have: [ A E D F ] = [ A B C ] [ B F D ] [ C E D ] 360 180 = 180 , [AEDF]=[\bigtriangleup ABC]-[\bigtriangleup BFD]-[\bigtriangleup CED]\le 360-180=180, and so [ D E F ] 180 2 = 90. [\bigtriangleup DEF]\le \frac{180}{2}=90. By the proof above, the equality case can be achieved when B D = D C = 15 BD=DC=15 .

Hence the answer is 90 90 .

Yi Hao Chan
May 20, 2014

Given the relationships in the question, we can deduce the following: AE = FD, AF = ED, common side EF, angle EFA = angle FED --> [DEF] = [AFE] FBD ~ EDC

Considering the trigonometric formula for triangles (0.5)(a)(b)(\sin \theta) when theta is the angle formed by adjacent sides Area of triangle is maximum when the ab\sin \theta is maximum.

Since base BC is divided into 2 lengths (BD, DC), and that FBD ~ EDC, [DEF] would be at maximum when BD = DC as this maximizes the lengths of individual triangles.

Incidentally, this condition results in the 4 triangles having equal areas (considering alternate angles, it can be deduced that angle AFE, FBD, EDC and FED are equal)

Area ABC = 0.5 * base * height = 360

Since maximum value of [DEF] occurs when the area of triangles AEF, FBD, FED and EDC are equal , maximum value = 360/4 = 90

Andy Chen
May 20, 2014

Suppose we let point G be on DE such that FG is parallel to BC. Let x be the length of BD. Since FB || GD and FG || BD, FBDG is a parallelogram, so FG = x.

Note that

[DEF] = [DGF] + [EFG]

[DEF] = (1/2 * x * (distance from E to FG)) + (1/2 * x * (distance from D to FG))

[DEF] = 1/2 * x * (distance from E to BD)

Since ED || AB, triangle CDE is similar to triangle CBA. Thus:

CD / CB = (distance from E to BC) / (distance from A to BC)

(30 - x) / 30 = (distance from E to BC) / 24

distance from E to BC = 24 * (30 - x) / 30 = 4 * (30 - x) / 5

[DEF] = 1/2 * x * (4 * (30 - x) / 5) = (2/5) * x * (30 - x)

[DEF] = (2/5) * (30x - x^2) = (2/5) * (225 - (225 - 30x + x^2))

[DEF] = (2/5) * (225 - (15 - x)^2)

To minimize 225 - (15 - x)^2, we let 15 - x = 0:

[DEF] = (2/5) * 225 = 90.

Adán Medrano
May 20, 2014

First, let us note the area of $ABC$ remains constant, as the measure of $BC$ and the height corresponding to itself remains constant.

The area of $ABC$ is $\frac{30\times 24}{2}=360$. Now, since $DE$ is parallel to $AF$ and $DF$ is parallel to $AE$, we have $AEDF$ is a parallelogram, and thus, the area of $DEF$ is equal to the area of $AEF$.

Then again, because of the parallel segments we mentioned previously, we may note

$\frac{AF}{AB}=\frac{DC}{BC}=\frac{CE}{AC}=1-\frac{AE}{AC}.$

Let $\frac{1}{2}+x=\frac{AF}{AB}$ and $\frac{AE}{AC}=\frac{1}{2}-x$ with $0\leq x\leq 1$. Let $s\left(XYZ\right)$ denote the area of triangle $XYZ$. We have

$\frac{s\left(AEF\right)}{s\left(ABC\right)}=\frac{AF\cdot AE\cdot \sin{\angle AEF}}{AB\cdot AC \cdot \sin{\angle BAC}}=\left(\frac{1}{2}+x\right)\left(\frac{1}{2}-x\right)=\frac{1}{4}-x^{2}\leq \frac{1}{4}$

and thus we have

$4s\left(AEF\right)\leq \left(ABC\right)=360$

so

$s\left(AEF\right)\leq 90$

and this can be achieved when $x=0$, when $D$ is midpoint of segment $BC$.

Sayeed Tasnim
May 20, 2014

By the Side Splitter Theorem, A B C F B D E D C \triangle ABC ~ \triangle FBD ~ \triangle EDC .

A formula for the area of a triangle is also [ A B C ] = 1 2 a b sin C [ABC] = \frac{1}{2} ab \sin C . Hence, if two triangles have an angle where the angle measures are the same and the sides adjacent to the first triangle are a a and a a and the sides adjacent to the second triangle are m a ma and n a na , the ratio of the area of the second triangle to the first triangle is m n mn

Let C D / B C = a CD/BC=a . Hence D B / B C = 1 a DB/BC=1-a , B F / A B = 1 a BF/AB=1-a , A F / A B = a AF/AB=a , C E / A C = a CE/AC=a , and A E / A C = 1 a AE/AC=1-a .

Hence, [ D E F ] = [ A B C ] [ B F D ] [ D E C ] [ F A E ] = [ A B C ] ( 1 ( 1 a ) 2 a 2 ( 1 a ) a ) = [ A B C ] ( a a 2 ) [DEF] = [ABC] - [BFD] - [DEC] - [FAE] = [ABC] (1 - (1-a)^2 - a^2 - (1-a)a) = [ABC] (a-a^2) .

a a 2 = ( a 1 2 ) 2 1 4 a-a^2 = -(a- \frac{1}{2})^2 - \frac{1}{4} so the maximum value of a a 2 a-a^2 is 1 4 \frac{1}{4} .

Hence, [ D E F ] = [ A B C ] ( a a 2 ) < = 1 4 [ A B C ] = 1 4 1 2 ( 24 ) ( 30 ) = 90 [DEF] = [ABC] (a-a^2) <= \frac{1}{4} [ABC] = \frac{1}{4} \frac{1}{2} (24)(30)=90 .

Joel Cardinal
May 20, 2014

Since DE || AB and FD || AC, ΔEDC ~ ΔFBD ~ ΔABC with base to height ratio 5:4. Also, AFDE is a parallelogram, thus, 2[DEF] = [AFDE].

In ΔEDC, let base CD = x with height 4 5 \frac {4}{5} x and area 2 5 x 2 \frac {2}{5}x^2 .

In ΔFBD, base DB = (30-x) with height 4 5 \frac {4}{5} (30-x) and area 2 5 ( 30 x ) 2 \frac {2}{5}(30-x)^2 .

[DEF] = 1 2 \frac {1}{2} [AFDE] = 1 2 \frac {1}{2} ( [ABC] - [EDC] - [FBD] )

= 1 2 \frac {1}{2} ( 1 2 \frac {1}{2} (30)(24) - 2 5 x 2 \frac {2}{5}x^2 - 2 5 ( 30 x ) 2 \frac {2}{5}(30-x)^2 )

= 180 - 1 5 x 2 \frac {1}{5}x^2 - 1 5 ( 900 60 x + x 2 ) \frac {1}{5}(900 - 60x + x^2)

= 2 5 ( 30 x x 2 ) \frac {2}{5}(30x - x^2)

= 2 5 ( 225 225 + 30 x x 2 ) \frac {2}{5}(225 - 225 + 30x - x^2)

= 2 5 ( 225 ( 15 x ) 2 ) \frac {2}{5}(225 - (15 - x)^2)

2 5 ( 225 ) \leq \frac {2}{5}(225)

= 90

Kevin Sun
May 20, 2014

We get that the area is 360. Because of similar triangles, FD/AC + DE/AB = 1. [DEF] = 1/2 DE DF sin(A), so we wish to maximize DE*DF, which is equivalent to maximizing (FD/AC)(DE/AB) because AC and AB are constant. By AM-GM, (FD/AC)(DE/AB) is maximized when (FD/AC)=(DE/AB)=1/2, so DEF is the medial triangle and [DEF] = 1/4 * 360 = 90.

Tobby Satyarama
May 20, 2014

AE//DF, AF//ED => AEFD is a //ogram, => [EFD] = 0.5[AEFD].

FBD is similar to ABC because FD//AC. Similarly, EDC is similar to ABC.

let DC/BC be a.

[EFD] = 0.5[AEFD]

= 0.5([ABC] - [EDC] - [FBD]) =0.5([ABC] - a^2[ABC] - (1-a)^2[ABC]) =0.5 ABC = ABC = ABC =< 0.25[ABC] = 90 sq. units

The Russian X
May 20, 2014

We can calculate [DEF] by doing [ABC]-[AFE]-[BFD]-[EDC]. [ABC] = 360 because 30 24/2=360. Assume length of BD is x, length of CD is 30-x. Then [BFD] = (x^2/900) 360 using ratios, [EDC] = ((900-60x+x^2)/900) 360, and [AFE] = (x(30-x)/900) 360. So [ABC]-[AFE]-[BFD]-[EDC] = [DEF] = 360-360 (x^2-30x+900)/900. Then taking the derivative gives 720x-10800=0, x=15. So the maximum value of [DEF] is 360-360(15^2-30 15+900)/900 = 360-360*675/900 = 360-270 = 90.

Vali Dobre
May 20, 2014

If the drawing is correct then the triangles CED and CAB are proportional with the ratio k = C D C B = E D A B k=\frac {CD}{CB} = \frac {ED}{AB} and the triangles BDF and BCA are proportional with the ratio 1 k = B D C B = F D A C 1-k=\frac {BD}{CB} = \frac {FD}{AC} .

The area of DEF is S = 1 2 D E D F s i n ( F D E ) S=\frac {1}{2}\cdot DE\cdot DF\cdot sin(FDE) and because AFDE is a paralelogram the angles FDE and BAC have the same measure. Using the equations from the parragraph above, S (area of DEF) is 1 2 k A B ( 1 k ) A C s i n ( B A C ) \frac {1}{2}\cdot kAB\cdot (1-k)AC\cdot sin(BAC) = k ( 1 k ) [ A B C ] = 1 2 k ( 1 k ) B C H k(1-k) \cdot [ABC]=\frac {1}{2} \cdot k(1-k)BC \cdot H and H is the distance from A to the line BC.

Now, all we have to do is to maximize k ( 1 k ) , 0 k 1 k(1-k), 0 \leq k \leq 1 . The maximum is reached for k = 0.5 k=0.5 and the maximum is 0.25. Replacing k with 0.5, BC with 30, H with 24, the result is S = 0.25 30 24 2 = 90 S=\frac{0.25 \cdot 30 \cdot 24}{2}=90

maximum area will be possible when D,E and f are midpoints of the sides BC,AC,AB respectively.therefore area =0.25\times 0.5\times 24\times 30=90

Jan J.
Jan 10, 2014

We use barycentric coordinates A = ( 1 , 0 , 0 ) , B = ( 0 , 1 , 0 ) , C = ( 0 , 0 , 1 ) , D = ( 0 , p , 1 p ) , E = ( 1 q , 0 , q ) , F = ( r , 1 r , 0 ) A = (1,0,0), B = (0,1,0), C = (0,0,1), D = (0,p,1 - p), E = (1 - q,0,q), F = (r,1 - r,0) , where p [ 0 , 1 ] p \in [0,1] and q , r q,r depend on p p , then A B = ( 1 , 1 , 0 ) \overrightarrow{AB} = (-1,1,0) A C = ( 1 , 0 , 1 ) \overrightarrow{AC} = (-1,0,1) E D = ( q 1 , p , 1 p q ) \overrightarrow{ED} = (q - 1,p,1 - p - q) F D = ( r , p + r 1 , 1 p ) \overrightarrow{FD} = (-r,p + r - 1,1 - p) The fact that D E A B DE \parallel AB and F D A C FD \parallel AC gives us q 1 p = 1 \frac{q - 1}{p} = -1 r 1 p = 1 \frac{-r}{1 - p} = -1 Thus r = 1 p r = 1 - p , q = 1 p q = 1 - p , this gives us [ D E F ] = [ A B C ] 0 p 1 p p 0 1 p 1 p p 0 = [ A B C ] ( 1 p ) p = 360 ( 1 p ) p [DEF] = [ABC] \cdot \begin{vmatrix} 0 & p & 1-p\\ p & 0 & 1-p\\ 1 - p & p & 0 \end{vmatrix} = [ABC] \cdot (1 - p)p = 360 \cdot (1 - p)p Hence we wish to maximize 360 ( 1 p ) p 360(1 - p)p , this is quadratic with roots 0 , 1 0,1 , hence it approaches maximum at their arithmetic mean p = 1 / 2 p = 1/2 , so the maximal area is 360 1 2 1 2 = 90 . 360 \cdot \frac{1}{2} \cdot \frac{1}{2} = \boxed{90} \text{.}

Sharky Kesa
Jan 4, 2014

If you draw a diagram of this problem, you find that δ D E F \delta DEF has 1 4 \frac {1}{4} of the area of δ A B C \delta ABC . Now, all you need to do is 24 × 30 × 1 2 × 1 4 24 \times 30 \times \frac {1}{2} \times \frac {1}{4} which gives a result of 90 90 .

Proof: Since D E A B DE || AB , and F D A C FD||AC , therefore by Thales Theorem, we can prove E F B C EF||BC . E F B C EF||BC and E D A B ED||AB , therefore F E D B FEDB is a parallelogram and F E D F B D \triangle FED \cong \triangle FBD . Similarily we can prove that all four triangles inside of A B C \triangle ABC are congruent. The area of congruent triangles is equal, so 4 × [ D E F ] = [ A B C ] [ D E F ] = 90 4\times [\triangle DEF] = [\triangle ABC] \implies [\triangle DEF] = 90 .

shaurya gupta - 7 years, 5 months ago

0 pending reports

×

Problem Loading...

Note Loading...

Set Loading...